1 Dona Warren, Department of Philosophy, The University … · Chapter 1 - Conditionals Dona...

63
Chapter 1 - Conditionals Dona Warren, Department of Philosophy, The University of Wisconsin – Stevens Point 1 CHAPTER 1 - CONDITIONALS Here, you’ll learn: How to understand complex sentences by symbolizing conditionals recognizing equivalent conditionals assessing conditionals How to assess arguments for validity / invalidity by establishing that arguments are valid using the O rule establishing that arguments are invalid by recognizing the fallacy of assuming the consequent Symbolizing Conditionals with One Connector I) Symbolizing with “If…. then….” Let’s start our study of conditionals by considering the following argument: 1. If Max is a poodle then Max is a dog. (premise) 2. Max is a poodle. (premise) 3. Max is a dog. (from 1 and 2) Lines 1, 2, and 3 are all statements because each of them is either true or false and because that’s just what statements are; statements are simply sentences that are either true or false, although we might not always know which it is. i Lines 2 and 3 of this argument are atomic statements because they have no parts that are themselves statements. Instead, atomic statements are composed of a subject (what the statement is about) and a predicate (what is being said about the subject). The subject of both of these sentences is “Max.” The predicate in the sentence on line 2 is “is a poodle” and the predicate of the sentence on line 3 is “is a dog.” 1. If Max is a poodle then Max is a dog. (premise) 2. Max is a poodle. (premise) 3. Max is a dog. (from 1 and 2) We’ll study the logic of atomic statements in (appropriately enough) predicate logic. i “2+2=4,” “The President of the United States is a Martian,” and “There is intelligent life on other planets,” are all statements, too. The first is true. The second is false. The third is either true or false, although I’m not sure which. “What time is it?” and “Please pass the sugar,” on the other hand, are not statements because they aren’t really true or false. Subject Predicate

Transcript of 1 Dona Warren, Department of Philosophy, The University … · Chapter 1 - Conditionals Dona...

Chapter 1 - Conditionals

Dona Warren, Department of Philosophy, The University of Wisconsin – Stevens Point 1

CHAPTER 1 - CONDITIONALS

Here, you’ll learn:

How to understand complex sentences by

• symbolizing conditionals

• recognizing equivalent conditionals

• assessing conditionals

How to assess arguments for validity / invalidity by

• establishing that arguments are valid using the → O rule

• establishing that arguments are invalid by recognizing the fallacy of assuming the consequent

Symbolizing Conditionals with One Connector

I) Symbolizing with “If…. then….”

Let’s start our study of conditionals by considering the following argument:

1. If Max is a poodle then Max is a dog. (premise)

2. Max is a poodle. (premise)

3. Max is a dog. (from 1 and 2)

Lines 1, 2, and 3 are all statements because each of them is either true or false and because that’s

just what statements are; statements are simply sentences that are either true or false, although

we might not always know which it is.i

Lines 2 and 3 of this argument are atomic statements because they have no parts that are

themselves statements. Instead, atomic statements are composed of a subject (what the statement

is about) and a predicate (what is being said about the subject). The subject of both of these

sentences is “Max.” The predicate in the sentence on line 2 is “is a poodle” and the predicate of

the sentence on line 3 is “is a dog.”

1. If Max is a poodle then Max is a dog. (premise)

2. Max is a poodle. (premise)

3. Max is a dog. (from 1 and 2)

We’ll study the logic of atomic statements in (appropriately enough) predicate logic.

i “2+2=4,” “The President of the United States is a Martian,” and “There is intelligent life on other planets,” are all

statements, too. The first is true. The second is false. The third is either true or false, although I’m not sure which.

“What time is it?” and “Please pass the sugar,” on the other hand, are not statements because they aren’t really true

or false.

Subject Predicate

Chapter 1 - Conditionals

Dona Warren, Department of Philosophy, The University of Wisconsin – Stevens Point 2

For now, we’ll represent atomic statements with capital letters. We can choose any letter we

want to represent an atomic statement, but we’ll make a practice of using the same letter for each

occurrence of the same statement in an argument and we won’t use the same letter to symbolize

different statements.

Letting “Max is a poodle” be “P,” and “Max is a dog” be “D,” we can rewrite our argument as

follows:

1. If P then D (premise)

2. P (premise)

3. D (from 1 and 2)

Line 1 is a compound statement because it has parts that are themselves statements. Compound

statements are constructed by joining two or more constituent statements together with statement

connectors. We’ll study the logic of such statements in propositional logic.

A compound statement composed of two statements joined by the statement connector

“if…then…” is called a conditional. The statement (simple or compound) that follows the “if” is

called the antecedent. The statement (simple or compound) that follows the “then” is called the

consequent.

Conditional

“If [Antecendent] then [Consequent].”

We’ll symbolize conditionals using an arrow.

Rule: “If ���� then �” is symbolized as “���� → �.”

Completely symbolized, then, our argument looks like this:

1. P → D (premise)

2. P (premise)

3. D (from 1 and 2)

Figuring Things Out

Symbolize the following conditionals and complete the following rule. The answers are in the

endnotes.

1. “If you’re a physics major then your major is physics.”1

2. “If you’re taking a physics class then you’re taking a science class.”2

Is this sentence true?3

Chapter 1 - Conditionals

Dona Warren, Department of Philosophy, The University of Wisconsin – Stevens Point 3

3. “If you’re taking a science class then you’re taking a physics class.”4

Is this sentence true?5

Rule: ���� → � (= or ≠) � → ����.6

II) Symbolizing with “If,” “Should,” and “Provided that”

Conditionals are extremely important – probably one of the most important kind of sentences

from a logical point of view.ii And because there are many different ways to express conditionals

besides the straightforward “If…then…,” we need to know how to recognize and symbolize

conditionals when they’re communicated through a variety of sentences.

Figuring Things Out

Symbolize the following conditionals and complete the following rules. The answers are in the

endnotes.

1. “If Jim is a pediatrician, he’s a doctor.”7

2. “Jim is a doctor, if he’s a pediatrician.”8

Rule: If ����, � = → .

Rule: ����, if � = → .

Rule: In general, whatever follows “if” becomes the of the conditional. 9

3. “Should the movie be good, we’ll recommend it.”10

4. “We’ll recommend the movie, should it be good.”11

5. “Provided that it’s a nice day, we’ll go on a walk.”12

6. “We’ll go on a walk, provided that it’s a nice day.”13

Rule: “Should” and “provided that” work exactly like and whatever follows them

becomes the of the conditional.14

ii Conditionals are important to logic because logic doesn’t give us any particular facts about the world. It doesn’t

tell us whether or not a certain animal is a poodle, for example, or whether or not a certain person is a bachelor.

What logic tells us is the relationships that would obtain between these facts if they were true. It tells us, for

example (and very roughly), that if an animals is a poodle then that animal is a dog, and that if a person is a bachelor

then that person is unmarried. We might go so far as to say that conditionals are the very heart of logic.

Chapter 1 - Conditionals

Dona Warren, Department of Philosophy, The University of Wisconsin – Stevens Point 4

Practice

Symbolize the following. The answers are in the endnotes.

1. “If C, D.”15

2. “A, if B.”16

3. “P, provided that Q.”17

4. “Provided that R, S.”18

5. “Should L, A.”19

6. “A, should P.”20

Rewrite the following conditionals by filling in the blanks:

1. “If the painting is expensive then he’ll like it.”21

“If , .”

“ , if .”

2. “If the class is open then we’ll enroll.”22

“Should , .”

“ , should .”

3. “If computers can think then minds are physical.”23

“Provided that , .”

“ , provided that .”

Chapter 1 - Conditionals

Dona Warren, Department of Philosophy, The University of Wisconsin – Stevens Point 5

III) Symbolizing with “Sufficient” and “Necessary”

There are two other important ways to express conditionals.

Figuring Things Out

Symbolize the following conditionals and complete the following rule. The answers are in the

endnotes.

1. “Getting 100% is a sufficient condition for passing the test.”24

(Hint: To say that something is sufficient is to say that it’s enough and maybe even more than

enough, as in “Having a trillion dollars is sufficient for being wealthy.” Certainly, having a

trillion dollars is enough – and more than enough - to count as wealthy so if someone has a

trillion dollars then that person is wealthy. Most wealthy people don’t have anywhere near a

trillion dollars. Same here. Getting 100% is enough – and more than enough – for passing the

test. Certainly anyone who got perfect score passed. And most people who passed probably

didn’t get a perfect score.)

2. “Getting 100% is sufficient for passing the test.”25

3. “To pass the test, it’s sufficient to get 100%.”26

Rule: Whatever is being described as sufficient becomes the of the

conditional.27

4. “Completing the foreign language requirement is a necessary condition for obtaining your

degree.”28

(Hint: To say that something is necessary is to say that it’s required, although other things might

be required too, as in “Fuel is necessary for the car to go.” Fuel is required for the car to go, so if

the car is going then it must have fuel, although other things – like a working engine – are

required too. Same here. Completing the foreign language requirement is required for the degree,

although other things – like completing the science requirement – are required too.)

5. “Completing the foreign language requirement is necessary for obtaining your degree.”29

6. “To obtain your degree, it’s necessary to complete the foreign language requirement.”30

Rule: Whatever is being described as necessary becomes the of the

conditional.31

Chapter 1 - Conditionals

Dona Warren, Department of Philosophy, The University of Wisconsin – Stevens Point 6

(I once had a student who had great difficulty remembering how to symbolize sentences that

used the terms “sufficient” or “necessary,” until she thought of it this way:

Symbolizing these sentences is Super Nasty. In the expression “Super Nasty,” you have “S” first,

then “N.” As in “S → N.” As in “sufficient conditions are antecedents and necessary conditions

are consequents.” ☺)

Practice

Symbolize the following. The answers are in the endnotes.

1. “Q is sufficient for D.”32

2. “For E, it’s sufficient that O.”33

3. “L is a sufficient condition for Q.”34

4. “A is necessary for B.”35

5. “R is a necessary condition for C.”36

6. “For T, it’s necessary that D.”37

Rewrite the following conditionals by filling in the blanks:

1. “If Robin is a surgeon then she’s smart.”38

“ is sufficient for .”

“ is necessary for .”

2. “If the power was out then the clocks will be blinking.”39

“ is a sufficient condition for .”

“ is a necessary condition for .”

3. “If ghosts exist then minds are nonphysical.”40

“ is necessary for .”

“ is sufficient for .”

Chapter 1 - Conditionals

Dona Warren, Department of Philosophy, The University of Wisconsin – Stevens Point 7

IV) Symbolizing with “Only if”

To ease into the logic of “only if,” suppose that George really likes shrimp. In fact, he likes

shrimp so much that “George will attend a party only if shrimp is being served” is true.

Now suppose we’re at a party. If we see George, can be we certain that there’s shrimp

somewhere? Is “if George is at a party then shrimp is being served” true? Yes, because George

attends parties only if shrimp is being served. George doesn’t attend any parties that don’t have

shrimp.

If we see shrimp, can we be certain that George is somewhere? Is “if shrimp is being

served then Georges is at the party” true? No, because maybe George wasn’t invited, or maybe

there are two simultaneous shrimp parties and George can’t attend both without bi-locating.

In summary, “George will attend a party only if shrimp is being served” is equivalent to

“If George is at a party then shrimp is being served.” It’s not equivalent to “If shrimp is being

served then Georges is at the party.”

Alternatively, let’s suppose that “Brenda is eligible to graduate only if she’s completed the math

requirement” is true.

Does it follow from this that “If Brenda is eligible to graduate then she’s completed the

math requirement” is true? Yes. She can graduate only if she’s completed the math requirement,

so if she can graduate then she must have completed the requirement.

Does it follow from this that “If Brenda has completed the math requirement then she’s

eligible to graduate?” No, because there are presumably many other graduation requirements,

and Brenda might not have met those yet.

In summary, “Brenda is eligible to graduate only if she’s completed the math

requirement” is equivalent to “If Brenda is eligible to graduate then she’s completed the math

requirement.” It’s not equivalent to “If Brenda has completed the math requirement then she’s

eligible to graduate.”

Rule: ���� only if � = → .

Rule: In general, whatever follows “only if” becomes the of the

conditional.41

It can also be helpful to realize that “if” introduces sufficient conditions and “only if” introduces

necessary conditions. (Think about that for awhile. If you’re not thinking too hard, that should

make perfect sense. If we say “She’ll accept the job offer only if benefits are included,” we’re

saying that the inclusion of benefits are necessary for her accepting the job offer.) As we’ve seen

above, necessary conditions become the consequents of conditionals. Therefore, the sentence

introduced by “only if” becomes the consequent of a conditional.

Note: The rules for “if” and “only if” refer to what follows “if” and “only if.” The rules for

“sufficient” and “necessary” refer to what is being described as sufficient and necessary.

Chapter 1 - Conditionals

Dona Warren, Department of Philosophy, The University of Wisconsin – Stevens Point 8

Practice

Symbolize the following. The answers are in the endnotes.

1. “Democracy will work only if the citizens are well-informed.” 42

2. “Only if the citizens are well-informed, can democracy work.”43

3. “You can join the club only if you pay the dues.”44

4. “Only if you play the dues, can you join the club.”45

5. “P only if Z.”46

6. “P, if Z.”47

7. “If L, M.”48

8. “Only if L, M.”49

Rewrite the following conditionals by filling in the blanks:

1. “If Jill enjoyed the movie then it must be a foreign film.”50

“ only if .”

“ , if .”

2. “If the cookie is good for you then it contains raisins.”51

“ , if .”

“ only if .”

3. “If Hank won then the election was rigged.”52

“ only if .”

“ , if .”

Chapter 1 - Conditionals

Dona Warren, Department of Philosophy, The University of Wisconsin – Stevens Point 9

V) Learning how to Symbolize: “Meaning” vs. “Rules”

We’ve developed rules for symbolizing conditionals expressed as

� “If ___ then ___”

� “If ___ , ___” and “___, if ___”

� “Should ___, ___” and “___, should ___”

� “Provided that ___, ___” and “___, provided that ___”

� “___ is a sufficient condition for ___” and “For ___ it’s sufficient that ___”

� “___ is a necessary condition for ___” and “For ___ it’s necessary that ___”

� “___ only if ___”, and “Only if ___, ___”

At this point, it’s natural to wonder whether it’s better to learn how to symbolize conditionals by

memorizing the rules or by understanding the meaning of the sentences. That’s a good question,

and the answer to it partly depends upon what you mean by “understanding the meaning of the

sentences.”

Generally speaking, it’s not a good idea to symbolize a compound sentence by thinking too much

about the meaning of its component sentences (the sentences that go in the blanks above). This is

because thinking too much about the component sentences can easily lead us to symbolize the

sentence in way that reflects what we think the sentence should say, rather than in a way that

reflects what the sentence actually says.

Suppose, for example, that we’re presented with the sentence “Being about to carry on an

intelligent conversation is necessary for having a mind.” If we reflect too much upon the

relationship between taking being able to carry on an intelligent conversation and having a mind,

we might think in the following mistaken way:

“I’m going to let ‘C’ be ‘Something can carry on an intelligent conversation’ and ‘M’ be ‘That

thing has a mind.’ Now I don’t think it’s the case that if something has a mind then it can carry

on an intelligent conversation. After all, I think that some nonhuman animals have minds even

though they can’t carry on a conversation. Therefore, I don’t think that ‘M → C’ is right. What

about ‘C → M?’ That says that if something can carry on an intelligent conversation then it must

have a mind. Yes! I agree with that. It would be impossible to carry on an intelligent

conversation without a mind. So ‘C → M’ must be the right answer. ‘Being able to carry on an

intelligent conversation is necessary for having a mind,” is symbolized as ‘C → M.’

Can you see what’s going wrong here? By thinking about the meaning of the component ideas,

we symbolized the relationship that we believe actually obtains between those ideas and not the

relationship that the sentence asserts to obtain between those ideas. (We might say that we’re

reading into the sentence, rather than reading the sentence as it stands.)

If we follow the rule for symbolizing sentences that employ the expression “necessary,” on the

other hand, we’d remember that the thing described as necessary becomes the consequent of the

conditional and we’d correctly symbolize ‘Being able to carry on an intelligent conversation is

necessary for having a mind,” as ‘M → C.’

Chapter 1 - Conditionals

Dona Warren, Department of Philosophy, The University of Wisconsin – Stevens Point 10

The moral of this story is that it’s better (i.e. more reliable) to symbolize a sentence by following

the rules than it is to symbolize a sentence by thinking about the meaning of the component

sentences.

But it can be unsatisfying to think that you need to memorize rules instead of understanding the

meaning or the reason behind the rules. After all, aren’t we always told that we should

understand rather than memorize? There are four responses to this concern, and I think that all of

them are right.

1) Remember that the symbolization rules didn’t just drop from the sky. Instead, we symbolized

sentences using the connector expressions and then generalized the rules from those

symbolizations.

This means that if you understood how to symbolize the original sentences from which we

generalized when formulating the rules, and if you saw how the rules simply represent those

generalizations, then you do understand the rules. You don’t need to reinvent the wheel each

time by figuring out the rule all over again each time you’re faced with a particular connector. Of

course you can reinvent the wheel, and you might want to, which brings us to…

2) We can symbolize sentences by understanding meanings rather than by memorizing rules, as

long as we remember that the meaning at issue is the meaning of the connector expressions, like

“necessary,” “sufficient,” “should,” “provided that,” and “only if,” and not the meaning of the

component sentences.

So, for example, one way to symbolize a sentence like “A only if B” is to think about how “only

if” works, possibly by remembering a favorite sentence like “Max is a poodle only if Max is a

dog.” You can then think about how you’d symbolize that sentence, conclude that it would be

symbolized as “P → D,” and transfer that insight to the sentence you need to symbolize. That’s a

perfectly acceptable approach to symbolization. It might take a little longer than simply applying

the rules, but if you’re more comfortable with this approach then by all means take it. Don’t,

however, adopt this approach to symbolizing because you think that memorizing the rules

somehow conflicts with understanding the connectors, because…

3) The meaning of the connectors just is how they work to ‘connect up’ ideas. Since the rules

express how the connectors work, to a very large extent understanding the meaning of a

connector just is knowing the symbolization rule. Understanding and memorization are not easily

separable here.

Let’s think again about how we might symbolize “A only if B” by working through a favorite

sentence like “Max is a poodle only if Max is a dog.” When we reflect on this sentence,

concluding that it would be symbolized as “M → D,” we’re using that favorite sentence to

remind us of how “only if” connects ideas. This is exactly the information that’s given in the rule.

And finally, relying on symbolization rules is acceptable because…

Chapter 1 - Conditionals

Dona Warren, Department of Philosophy, The University of Wisconsin – Stevens Point 11

4) Although we can memorize things without understanding them, and so memorization and

understanding are not always the same thing, it’s frequently a mistake to think of memorization

as the enemy of understanding because sometimes we can fully understand something only after

we’ve memorized it.

If that makes perfect sense to you because you can remember some occasions in which your

learning worked that way, you can stop reading this section and proceed to practice symbolizing.

If, however, you’re skeptical about this, think for a moment about how you learned to multiply

26 by 3. I’ll bet you learned to do it like this: First you multiply 6 by 3 to get 18. You then “carry

the 1” which you add to what you get when you multiply 2 by 3. And this gives you the answer

of 78. It looks like this:

1

26

x 3

78

You memorized this procedure, I’ll wager, without really understanding why you carry the 1,

and you might follow the procedure now without ever thinking about why you carry the 1.

Of course, there is a reason. Here it is: 26 means that you have 2 groups of ten and 6 groups of

one. When you multiply the 6 groups of one by 3, you have 18 groups of one. In other words,

you have 1 group of ten and 8 groups of one. Remember that because we’ll refer to it later. Now

you need to multiple your 2 groups of ten by 3. This gives you 6 groups of ten. But you still have

that 1 group of ten from before, so in all you have 7 groups of ten and the 8 groups of one. 7

groups of ten and 8 groups of one is 78. We might represent it like this:

26 x 3 =

(2 tens + 6 ones) x 3 =

(2 tens x 3) + (6 ones x 3) =

(2 tens x 3) + (18 ones) =

(2 tens x 3) + (1 ten and 8 ones) =

(6 tens) + (1 ten and 8 ones) =

7 tens + 8 ones =

78

Carrying the 1 is just a way of keeping track of the fact that we get one extra unit of ten when we

multiple 6 by 3. It’s a book-keeping technique.

Sometimes children are taught how to multiply in this way. In fact, I’ll bet that your math

textbook had an explanation something like the one I just gave you. It was probably

accompanied by pictures of blocks. And there might be children who completely understand that

explanation and think about the reasoning behind carrying the 1 (or the 2 or the 3) each and

every time they do it. But I don’t think that most children are like that. I suspect that most

children find the explanation confusing and just learn to carry the 1 (or the 2 or the 3). If, many

Chapter 1 - Conditionals

Dona Warren, Department of Philosophy, The University of Wisconsin – Stevens Point 12

years later, they take a course in college designed for elementary school teachers, they might

learn the explanation for why people carry the 1 and have a pleasurable “Ah-ha” moment. Why

didn’t they have that moment when they were learning how to multiply in elementary school?

It’s because understanding why they carry the 1 makes a lot more sense after they’ve memorized

that they should carry 1 and have internalized the procedure with a lot of examples. The same

applies here. If you don’t completely understand a symbolization rule, don’t let that deter you

from memorizing it. You might find yourself understanding it quite well after you’ve completely

internalized the rule through memorization and lots of practice.

Practice

Symbolize the following sentences, letting B be “Behaviorism is right” and D be “Mental states

are dispositions to behave.” The answers are in the endnotes.

1. “If behaviorism is right then mental states are dispositions to behave.”53

2. “Behaviorism is right only if mental states are dispositions to behave.”54

3. “For behaviorism to be right, it’s sufficient that mental states be dispositions to behave.”55

4. “Should behaviorism be right, mental states are dispositions to behave.”56

5. “Behaviorism is right, provided that mental states are dispositions to behave.”57

6. “Mental states being dispositions to behave is a necessary condition for behaviorism being

right.”58

7. “Behaviorism is right, if mental states are dispositions to behave.”59

8. “Only if mental states are dispositions to behave, is behaviorism right.”60

9. “Behaviorism is right, should mental states be dispositions to behave.”61

10. “For mental states to be dispositions to behave, it’s necessary that behaviorism is right.”62

Recognizing Equivalent Conditionals

We’ve seen that a sentence like “If Max is a poodle then Max is a dog” is not the same as a

sentence like “If Max is a dog then Max is a poodle.” (The first sentence is true because all

poodles are dogs. The second sentence is false because not all dogs are poodles.) Order matters

very much when we have conditionals.

“If Max is a poodle then Max is a dog” is equivalent to many other sentences, though, including

• “Max is a dog, if Max is a poodle.”

• “Max is a poodle only if Max is a dog.”

• “Should Max be a poodle, Max is a dog.”

Chapter 1 - Conditionals

Dona Warren, Department of Philosophy, The University of Wisconsin – Stevens Point 13

• “Provided that Max is a poodle, Max is a dog.”

• “Max being a poodle is sufficient for Max being a dog.”

• “Max being a dog is necessary for Max being a poodle.”

To see that all of these sentences are equivalent, we can notice that they’re all symbolized in the

same way. They’re all symbolized as “P → D.”

In general, that’s a good way to see if two sentences say the same thing. Just symbolize the

sentences and see if the symbolizations are the same (or, later on, see if the symbolizations are

equivalent to each other). If so, then the sentences say the same thing. If not, then they don’t.

Practice

Which of the following sentences are equivalent to

“If our actions have moral value then we have free will.”? The answers are in the endnotes.

1. “Our actions have moral value only if we have free will.”63

Equivalent / Not equivalent

2. “Our actions have moral value, if we have free will.”64

Equivalent / Not equivalent

3. “Our actions have moral value, provided that we have free will.”65

Equivalent / Not equivalent

4. “Our actions having moral value is a sufficient condition for our having free will.”66

Equivalent / Not equivalent

5. “Our having free will is a necessary condition for our actions having moral value.”67

Equivalent / Not equivalent

Which of the following sentences are equivalent to

“Having a shape is a necessary condition for having a color”? The answers are in the endnotes.

1. “Something has a shape only if it has a color.”68

Equivalent / Not equivalent

2. “Something has a color only if it has a shape.”69

Equivalent / Not equivalent

3. “Something has a shape if it has a color.”70

Equivalent / Not equivalent

4. “Having a color is a sufficient condition for having a shape.”71

Equivalent / Not equivalent

5. “Something has a color, provided that it has a shape.”72

Equivalent / Not equivalent

Chapter 1 - Conditionals

Dona Warren, Department of Philosophy, The University of Wisconsin – Stevens Point 14

Assessing Conditionals

I) What Makes a Conditional False

When is a conditional true and when is it false? What are the truth conditions for conditionals?

Philosophers and logicians treat all conditionals as material conditionals. Material conditionals

are like promises. The conditional is false when (and only when) the promise is broken.

Consider the following (pseudo!) promise: “If you clean my house then you’ll get an A in my

class.” What would need to happen for this promise to be broken? Well, you’d have to clean my

house, holding up your end of the bargain, without me giving you an A in return. That’s the only

way that I could break my promise. If you do clean my house and I give you A, obviously I

haven’t broken my promise. And if you don’t clean my house then, in some sense, the promise

doesn’t apply because I only said what I’d do if you did clean. If you don’t clean my house, I’m

free to give you an A or not. Neither decision will break my promise, because the promise isn’t

in force, so to speak. We can represent this information in the following chart:

You clean my house. You got an A. If you clean my house then

you’ll get an A in the class.

Yes Yes Kept

Yes No Broken

No Yes Kept

No No Kept

And we can generalize from this to represent the truth conditions for a conditional in a truth

table.

���� � ���� → �

T T T

T F F

F T T

F F T

Rule: A conditional is false when and only when the antecedent is

and the consequent is .73

By the way, the truth table defines the meaning of “→” and, in so doing, it gives us what is

called “the material conditional.” There is some “slippage” between the material conditional and

many “if…then…” sentences that we encounter in real life. For example, “If the Pope won the

lottery then he’d spend the money on fast cars” is intuitively false (because fast cars aren’t the

sort of thing we’d expect the Pope to spend his money on) but it’s a true material conditional

(because the antecedent is false; the Pope didn’t win the lottery). For our purposes, however, we

Chapter 1 - Conditionals

Dona Warren, Department of Philosophy, The University of Wisconsin – Stevens Point 15

don’t need to worry about this too much because the material conditional captures enough of the

logical features of normal conditionals to allow us to properly the evaluate arguments that

contain conditionals.

II) Assessing Conditionals

The ability to identify the “falsification conditions” for conditional statements is very useful

because many important claims are conditional in nature. This is especially true in philosophy.

By understanding how conditional claims can be proven false, we’re in a much better position to

level appropriate objections to such claims.

For example, let’s consider the sentence “If we’re hallucinating all the time then we can’t know

anything.”

This sentence is false if the antecedent, “We’re hallucinating all the time,” can be true

while the consequent, “We can’t know anything,” is false. Of course, if it’s false that we can’t

know anything, then it must be true that we can know something. (If that confuses you, you’re

just thinking too hard.)

Thus, the falsification condition for “If we’re hallucinating all the time then we can’t

know anything” is “We are hallucinating all the time (true antecedent) + We can know

something (false consequent),” and this condition can be expressed in a number of ways, such as

“Even if we’re hallucinating all the time, we might know some things, like math,” and “We can

know certain things, like logical truths, even if we’re always hallucinating.” (It doesn’t matter

whether we mention the true antecedent or the false consequent first. All that matters is that

we’re describing a situation in which the antecedent is true and the consequent is false.)

It’s easy to think “If we’re hallucinating all the time then we can’t know anything” would

also be proven false by “Maybe we don’t know anything even if we’re not hallucinating all the

time” but this is mistaken because that circumstance would be one in which the antecedent of the

conditional is false and the consequent is true. (Don’t believe me? Check it out for yourself.

Better yet, check it out for yourself even if you do believe me.)

Now let’s consider “The existence of God is a necessary condition for a well-designed universe.”

Because this conditional isn’t expressed in straightforward “if… then…” form, the first thing we

should do is symbolize this sentence in order to make sure that we correctly identify the

antecedent and the consequent.

Letting “G” be “God exists,” and “W” be “The universe is well-designed,” “The

existence of God is a necessary condition for a well-designed universe” is symbolized as

“W → G.” This allows us to see that “The universe is well-designed” is the antecedent of this

conditional and “God exists,” is the consequent.

Thus, the falsification condition for “The existence of God is a necessary condition for a

well-designed universe” is “The universe is well-designed (true antecedent) + God doesn’t exist

(false consequent),” and we can express this in a number of ways, including “The universe might

be well-designed even if God doesn’t exist,” and “Purely naturalistic forces could account for the

design that we see in the universe.” (It’s true that this second way of expressing the falsification

condition doesn’t explicitly say “God doesn’t exist,” but it accounts for how the universe could

be well-designed even if God didn’t exist so “God doesn’t exist” is still there, sort of hovering in

the background.)

Chapter 1 - Conditionals

Dona Warren, Department of Philosophy, The University of Wisconsin – Stevens Point 16

If someone said, “Why would God create a well-designed universe?” however, she

wouldn’t be leveling a relevant objection to the conditional because this question alludes to the

fact that the antecedent might be false while the consequent is true.

It’s very easy to be mistaken about what would, and what wouldn’t, show a conditional to be

false, and many people get it wrong more often than not. This is why the ability to correctly

identify falsification conditions is a skill worth obtaining. Knowing how to properly object to

conditional statements, how to recognize improper objections, will give you a real advantage in

many contexts.

Practice

For each of the claims below identify the objection that, if true, would show that claim to be false.

1. “If we can’t know anything for certain then knowledge isn’t worth seeking.”74

a. Although we can’t know anything for certain, knowledge is worth seeking.

b. Although we can know some things for certain, knowledge isn’t worth seeking.

2. “If a belief is useful then we’re justified in holding it.”75

a. We’re justified in holding some beliefs even though they’re not useful.

b. We’re not justified in holding some beliefs even though they are useful.

3. “If euthanasia maximizes happiness then it’s ethical.”76

a. Some things that maximize happiness aren’t ethical.

b. Some things that are ethical don’t maximize happiness.

4. “Lying is unethical only if it violates our culture’s moral code.”77

a. Lying is unethical even if it doesn’t violate our culture’s moral code.

b. Lying violates our culture’s moral code even if it isn’t unethical.

5. “Provided that some beliefs are justified without argument, religious faith is philosophically

acceptable.”78

a. Even though no beliefs are justified without argument, religious faith is philosophically

acceptable.

b. Even though some beliefs are justified without argument, religious faith isn’t philosophically

acceptable.

6. “God doesn’t know our future actions, if we have free will.”79

a. “God knows our future actions but we have free will anyway.”

b. “God doesn’t know our future actions but we don’t have free will anyway.”

Chapter 1 - Conditionals

Dona Warren, Department of Philosophy, The University of Wisconsin – Stevens Point 17

7. “We have good reason to believe in God, provided that we have good reason to believe in

miracles.”80

a. Even though we don’t have good reason to believe in God, we do have good reason to believe

in miracles.

b. Even though we have good reason to believe in God, we don’t have good reason to believe in

miracles.

8. “Cross-cultural agreement about ethics is a necessary condition for the objectivity of

morality.”81

a. Although morality is objective, there can still fail to be cross-cultural agreement about ethics.

b. Even if there is cross-cultural agreement about ethics, morality can still fail to be objective.

9. “For an action to be moral it’s sufficient that it satisfy Kant’s Categorical Imperative.”82

a. An action can be moral without satisfying Kant’s Categorical Imperative.

b. An action can satisfy Kant’s Categorical Imperative without being moral.

Symbolizing Conditionals with Multiple Connectors

Consider the following assignments to sentence letters.

M = You’re a philosophy major.

S = You’ll take symbolic logic.

E = You’ll enjoy the course.

What do the following sentences mean? Do they mean the same thing?

(M → S) → E

M → (S → E)

I find it pretty hard to get a grip on what these sentences are saying, so the surest way for me to

decide if these sentences mean the same thing is to construct their truth tables and see if they

differ.

M S E (M → S) → E M → (S → E)

T T T t T t t T t

T T F t F f t F f

T F T f T t t T t

T F F f T f t T t

F T T t T t f T t

F T F t F f f T f

F F T t T t f T t

F F F t F t f T t

Chapter 1 - Conditionals

Dona Warren, Department of Philosophy, The University of Wisconsin – Stevens Point 18

Because these sentences do differ in truth value on the sixth and eighth lines, (M → S) → E isn’t

the same thing as M → (S → E), which means that M → S → E is genuinely ambiguous. We’ll

always need to provide grouping symbols in a sentence like that.

The main connector in a symbolized sentence is the connector that stands outside the grouping

symbols. A well formed formula (wff) must be such that, when it’s decomposed around its main

connector, the remaining parts have only one main connector.

For example, the connectives in the following formula are ranked like this:

{A → [(B → C) → D]} → F

Of course, it’s one thing to identify the main connector in an already-symbolized sentence. It’s

quite another thing to identify the main connector in a natural-language sentence. That can be

very tricky indeed, until you get the hang of it. Being able to recognize the main connector in

natural language sentences is an important skill, though, because it can really help you to

understand the logically complex sentences that you find in places like philosophy and law.

How to find the main connector in an English sentenceiii

:

i) Identify all of the connectors. Underline or circle them, if you wish.

ii) For each connector, see what it connects.

iii) If there is nothing left over, that connector may be the main connector. If there is part of the

sentence left over, that connector isn’t the main connector.

Once we’ve identified the main connector, we symbolize “around” that main connector.

Let’s see how this works by doing some examples.

iii

If you encounter significant difficulties identifying the main connectors, here’s another approach for you to try:

i) Identify all of the connectors. Underline or circle them, if you wish.

ii) Read around each connector. Don’t leave out any words in the sentence but don’t say the connector you’re testing.

iii) If the things connected are sensible statements on their own (possibly after grammatical changes from the

subjunctive or gerund), then that connector may be the main connector. If the things connected aren’t sensible

statements on their own, then that connector isn’t the main connector.

1. Main Connector 2. Next to Main

3. Next to Next to Main Connector

4. Least Main Connector

Chapter 1 - Conditionals

Dona Warren, Department of Philosophy, The University of Wisconsin – Stevens Point 19

Example 1

“If this novel worth is reading then having an attractive cover is sufficient for being a good

book.”

First note the connectors:

“If this novel worth is reading then having an attractive cover is sufficient for being a good

book.”

Then identify the main connector:

“If… then…” connects “this novel worth is reading” and “having an attractive cover is sufficient

for being a good book.” Because there is nothing left over, that’s the main connector. (In contrast,

“is sufficient for” connects “has an attractive cover” with “is a good book” and “this novel is

worth reading” is left over.)

“If this novel worth is reading then having an attractive cover is sufficient for being a good

book.”

Finally, symbolize around the main connector:

“If (this novel worth is reading) then (having an attractive cover is sufficient for being a good

book.)”

“(this novel worth is reading) → (having an attractive cover is sufficient for being a good

book.)”

“R→ (having an attractive cover is sufficient for being a good book.)”

“R→ (C is sufficient for B.)”

“R→ (C → B)”

Chapter 1 - Conditionals

Dona Warren, Department of Philosophy, The University of Wisconsin – Stevens Point 20

Example 2

“Being a good bowler is necessary for membership only if it’s a bowling club.”

First note the connectors:

“Being a good bowler is necessary for membership only if it’s a bowling club.”

Then identify the main connector:

“Only if” connects “Being a good bowler is necessary for membership” with “it’s a bowling

club.” There’s nothing left over, so that’s the main connective.

“Being a good bowler is necessary for membership only if it’s a bowling club.”

Finally, symbolize around the main connector:

“(Being a good bowler is necessary for membership) only if (it’s a bowling club).”

“(Being a good bowler is necessary for membership) → C.”

“(B is necessary for M) → C.”

“(M → B) → C.”

Chapter 1 - Conditionals

Dona Warren, Department of Philosophy, The University of Wisconsin – Stevens Point 21

Example 3

“George will be admitted to the club if he seeks admission, provided that being a good bowler is

sufficient for membership.”

First note the connectors:

“George will be admitted to the club if he seeks admission, provided that being a good bowler is

sufficient for membership.”

Then identify the main connector:

“George will be admitted to the club if he seeks admission, provided that being a good bowler is

sufficient for membership.”

Finally, symbolize around the main connector:

“(George will be admitted to the club if he seeks admission,) provided that (being a good bowler

is sufficient for membership).”

“(G if A,) provided that (B is sufficient for M)”

“(B is sufficient for M) → (G if A)”

“(B → M) → (G if A)”

“(B → M) → (A → G)”

Chapter 1 - Conditionals

Dona Warren, Department of Philosophy, The University of Wisconsin – Stevens Point 22

Practice

Symbolize the following complex conditionals using the assigned sentence letters. The answers

are in the endnotes.

1. “If it’s the case that computers deserve moral consideration if they can think, then the ability

to think is the source of our moral value.”83

C = Computers can think

S = The ability to think is the source of our moral value.

M = Computers deserve moral consideration.

2. “If computers can think then provided that the ability to think is the source of our moral value,

computers deserve moral consideration.”84

C = Computers can think

S = The ability to think is the source of our moral value.

M = Computers deserve moral consideration.

3. “Provided that being able to carry on a conversation is sufficient for being able to think, the

Turing Test is an accurate indicator of artificial intelligence.”85

C = x can carry on an intelligent conversation.

T = x can think.

A = The Turing Test is an accurate indicator of artificial intelligence.

4. “Being able to carry on an intelligent conversation is necessary for having a mind only if many

people don’t have minds.”86

C = x can carry on an intelligent conversation.

M = x has a mind.

D = Many people don’t have minds.

5. “If our being able to think is sufficient for our having moral value, then if computers can think,

they deserve moral consideration.”87

T = We can think.

V = We have moral value.

C = Computers can think.

M = Computers deserve moral consideration.

6. “Being able to think is necessary for having moral value only if being able to think is

sufficient for having a soul.”88

T = x can think.

V = x has moral value.

S = x has a soul.

Chapter 1 - Conditionals

Dona Warren, Department of Philosophy, The University of Wisconsin – Stevens Point 23

7. “Should being able to carry on an intelligent conversation be sufficient for having a mind, then

an answering machine would have a mind if it could answer back.”89

C = x can carry on an intelligent conversation.

M = x has a mind.

A = An answering machine has a mind.

B = An answering machine can answer back.

8. “If a computer could carry on an intelligent conversation then provided that being able to carry

on an intelligent conversation is sufficient for having a mind, the computer would have a mind.”

C = A computer can carry on an intelligent conversation.90

M = x has a mind.

I = x can carry on an intelligent conversation

H = A computer has a mind.

9. “Neither side can avoid circularity. This is disturbing if a non-question-begging way of

mutually resolving basic disagreements over the overall force of a body of evidence is a

necessary condition of the possibility of being rational in these matters.” (William J. Wainwright,

Reason and the Heart, Cornel University Press, 1995, p. 123)91

Symbolize the second sentences in this quotation, using the following sentence letters:

D = It’s disturbing that neither side can avoid circularity.

N = There is a non-question-begging way of mutually resolving basic disagreements over the

overall force of a body of evidence.

R = One can be rational in matters involving basic disagreements over the overall force of a body

of evidence

INFERENCE RULE: → O

Now that we know quite a bit about conditionals, we can study their logic by reflecting upon

what can, and what can’t, be inferred from them. Let’s start by reflecting upon one of my

favorite arguments:

1. If Max is a poodle then Max is a dog. (premise)

2. Max is a poodle. (premise)

3. Max is a dog. (from 1 and 2)

The inference in this argument is valid because if the premises are true then the conclusion must

be true as well. We can see this using the Bob Method. If Bob believes “If Max is a poodle then

Max is a dog” and if Bob believes “Max is a poodle,” Bob will be forced to believe “Max is a

dog.”

Now, what if instead of “Max,” we used the name “Leo?” Would the inference still be valid?

Sure.

And what if instead of “poodle,” we said “schnauzer?” Would the inference still be valid? Of

course.

Chapter 1 - Conditionals

Dona Warren, Department of Philosophy, The University of Wisconsin – Stevens Point 24

And what if instead of “Max is a poodle” we said “The book is over 500 pages” and instead of

“Max is a dog” we said “Sam will never finish reading it.” Would the inference still be valid?

Yes.

What makes this inference valid is the form of the sentences, not the content of the sentences.

In general, any inference of the following form will be valid:

1. → (Premise)

2. (Premise)

3. (from 1 and 2)

Since this kind of inference is both valid and very common, it’s worthwhile to take it as an

inference rule. Inference rules are argument forms that specify correct inferences. They tell us

what we may correctly infer, given certain kinds of information, and so they’re very useful.

We’ll call this inference rule “Arrow Out” (abbreviated “→O”) because it takes us from (or out

of) an arrow statement. Arrow Out tells us that from a line of the form “� → �” and a line of

the form “�” we can infer a line of the form “�.”

Sometimes it will be convenient to present an argument horizontally, with all of the premises on

the left, separated by commas, and the conclusion on the right, separated from the premises with

the symbol “|-.” For instance, the argument “� → �. �. Therefore �,” can be written like this:

“� → �, � |- �.” Because this way of representing arguments is so visually clean, I’ll use it to

present the inference forms. Arrow Out, for example, is presented as follows:

→O: ���� →→→→ ����, ���� |- ����

Before we start to actually use this rule, there are a couple of things that I want to mention.

• Why We’ll Use Shapes

First, many logic books use letters like “P” and “Q” when they present argument forms, instead

of using shapes like “�” and “�.” Such a book would present →O as “P → Q, P |- Q”.

I’ll be using shapes when I give you inference rules because I want to stress that the parts of the

compound sentence at issue (in this case the “�” and the “�” in the conditional “� → �”)

don’t need to be atomic statements symbolized by one letter but can, themselves, be compound

statements. We can think of these statements being written “inside” the shapes.

Chapter 1 - Conditionals

Dona Warren, Department of Philosophy, The University of Wisconsin – Stevens Point 25

So, Arrow Out doesn’t just tell us that from → and we can infer .

It also tells us that from → and we can infer .

Thinking in terms of shapes in which smaller formulas can be written helps us to develop the

habit of proper “mental clumping.” In the argument above, for instance, it allows us to see

“A → B” and “C → D” as basic clumps of information. This mental clumping makes logic much

easier.

• Primitive vs. Derived Inference Rules

Second, Arrow Out is what we’ll call a primitive inference rule. Many textbooks justify

primitive inference rules by appealing to a truth table. To justify Arrow Out in this way, we

construct a truth table that represents the premises and the conclusion of the argument. Then we

note that all of the lines of this truth table on which the premises are true must also represent the

conclusion as true. Like this:

Basic Term Basic Term Premise Premise Conclusion

���� � ���� → � ���� �

T T T T T

T F F T F

F T T F T

F F T F F

The first row is the only row in which both premises are true, and in that row the conclusion is

true as well. This demonstrates that whenever the premises are true, the conclusion must be true.

In other words, it demonstrates that the argument is valid.

Now I don’t know about you, but I don’t find this appeal to truth tables to be very helpful. I

understand it, but psychologically speaking, it doesn’t add anything. It doesn’t scratch any itch

or clear anything up because I don’t need a truth table to demonstrate to me that Arrow Out is

valid. Of course, it’s valid. You can just see that it’s valid. If we know that P gets us Q (i.e. if we

know “P→Q”) and if we know that we have P, then we’re bound to know that we have Q as well.

If you didn’t already think this way, you certainly would have perished long ago. Somewhere

along the line, you’d have thought something like “If the truck is coming then I should stand on

the curb,” and “The truck is something,” but you’d have failed to infer “I should stand on the

curb.” And if you failed to draw that inference, you’d have stepped out into traffic and you

wouldn’t be reading this now. We might say that the validity of Arrow Out is self evident.

Indeed, if someone didn’t understand Arrow Out, it would be practically impossible to find

anything simpler in terms of which Arrow Out could be explained to him. To that extent, this

inference rule is psychologically primitive.

P Q P Q

(A→B) (A→B) (C→D) (C→D)

Chapter 1 - Conditionals

Dona Warren, Department of Philosophy, The University of Wisconsin – Stevens Point 26

We’ll be getting other primitive inference rules as we go along, but I won’t bother to justify them

with truth tables because once we understand these rules it will be fairly easy for us to see that

they’re valid. Indeed, the fact that the validity of a rule is fairly easy to spot is what leads us to

take it as a primitive rule in the first place.

Not all valid inference rules are like this, though. Some valid inferences might not seem valid to

us at first. In this case, we’ll justify the validity of the rule by showing how it follows from other

rules that we already know to be valid. Inference rules that are justified in this way are called

derived inference rules.

In summary, then, some of our inference rules (the primitive rules) will be taken for granted

because they’re obviously valid, and the rest of our inference rules (the derived rules) will be

justified by appeal to rules that we already know to be valid. iv

ESTABLISHING VALIDITY: FORMAL PROOFS

Now that we have an valid inference rule in hand, we can take a look at the heart of symbolic

logic – formal proofs!

The basic idea is pretty simple. If we can proceed from the premises of an argument to its

conclusion by a series of valid inference rules (so far, we have only one inference rule), then the

argument as a whole is valid because all of its little steps are valid. This is the method of natural

deduction, otherwise known as the construction of formal proofs or derivations, for proving

validity.

A proof for an argument will symbolize all sentences in the argument and record them in a

numbered list, with the premises at the top and the conclusion at the bottom. (I usually make a

note of the conclusion to the far right of the last premise, as “want…,” to remind me of what I

want to prove. This isn’t really part of the proof, and it isn’t strictly necessary, but it will be very

helpful later on!) Each line in a proof is justified in the justification column on the right.

Premises get their justification for free and are noted with “P.” The justification for a derived

line and refers to the lines used and the rule employed in the derivation of that line. Here’s a little

baby proof of the argument P→Q, P |- Q:

1. P → Q P

2. P P - want Q

3. Q 1, 2 →O

(Isn’t it cute?)

iv “Fairly easy to see as valid” will be a necessary condition for being a primitive rule but not a sufficient condition.

In other words, if we take an inference as primitive then it must be fairly easy to see as valid (generally speaking),

but it isn’t the case that every inference that we can easily see as valid will be taken as primitive. Some fairly “easy”

inference rules will derived instead. This is because logicians generally try to take as few primitive inference rules as

possible. There’s more “elegant simplicity” to a system that takes two primitive inference rules and derives the rest

than there is to a system that takes, say 27 inference rules as primitive.

Chapter 1 - Conditionals

Dona Warren, Department of Philosophy, The University of Wisconsin – Stevens Point 27

Before you start constructing proofs of your own, there are some things that we should note:

Note: The order in which the premises appear never matters.

So this proof is fine.

1. P → Q P

2. P P - want Q

3. Q 1, 2 →O

And this proof is fine, too:

1. P P

2. P → Q P - want Q

3. Q 1, 2 →O.

We don’t have to have the “�→�” premise before we have the “�” premise.

Note: The order in which we write the down the numbers in the justification column never

matters.

Look at the proof we’ve just seen, we can write “1, 2 →O” in the justification column for line 3,

like so:

1. P P

2. P → Q P - want Q

3. Q 1, 2 →O.

Or we can write “2, 1 →O” in the justification column for line 3, like this:

1. P P

2. P → Q P - want Q

3. Q 2, 1 →O.

All that matters is that we refer to the correct lines, in this case lines 1 and 2; the order in which

we refer to these lines is immaterial. (I will follow the convention of always referring to the lines

in the order that they apper in the proof, so I’ll always write down “1, 2” rather than “2, 1,” but

that’s just my convention and nothing hangs on it.)

Chapter 1 - Conditionals

Dona Warren, Department of Philosophy, The University of Wisconsin – Stevens Point 28

Note: → O can be used only on two lines at a time. It’s a “two line rule.”

For example, here’s how we’ll construct a proof for this argument:

A, A → B, B → C, C → D | - D

First we’ll write down the premises, noting the conclusion that we want:

1. A P

2. A → B P

3. B → C P

4. C → D P - want D

Next, we’ll conclude B from lines 1 and 2 using →O:

1. A P

2. A → B P

3. B → C P

4. C → D P - want D

5. B 1, 2 →O

Then we’ll conclude C from lines 3 and 5 using →O:

1. A P

2. A → B P

3. B → C P

4. C → D P - want D

5. B 1, 2 →O

6. C 3, 5 →O

And finally, we’ll conclude D from lines 4 and 6 using →O:

1. A P

2. A → B P

3. B → C P

4. C → D P - want D

5. B 1, 2 →O

6. C 3, 5 →O

7. D 4, 6 →O

It might be tempting to put lines 1, 2, 3 and 4 together all at once, but that’s against the rules.

→O puts sentences together two at a time, as we did above.

Chapter 1 - Conditionals

Dona Warren, Department of Philosophy, The University of Wisconsin – Stevens Point 29

Note: Any statement, simple or compound, may be the antecedent, �, or the consequent, �.

For example, we can construct a proof for this argument: P → Q, (P → Q) → (R → S) |- R → S

1. P → Q P

2. (P → Q) → (R → S) P – want R → S

3. R → S 1, 2 →O

Here’s where mental clumping comes in handy. If you have trouble following that proof,

imagine the shapes around the components, like so:

1. P → Q P

2. (P → Q) → (R → S) P – want R → S

3. R → S 1, 2 →O

See? No big deal. Just think about the “P→Q” and the “R→S” as one unit.

Note: You can apply →O only to the main connector! (This will be true of all “out rules.”)

This point is incredibly important, and failing to fully appreciate it is the cause of many incorrect

proofs.

For example, consider the argument (A → C) → (B → D), A → C, B |- D

The following proof of this argument is incorrect.

1. (A → C ) → (B → D) P

2. A → C P

3. B P - want D

4. D 1, 3 →O

Can you see the problem? The proof fails on line 4 because →O is incorrectly applied. This

proof “arrows out” on the arrow between B and D, but that arrow isn’t the main connector on

line 1.

1. (A → C ) → (B →→→→ D) P

2. A → C P

3. B P - want D

4. D 1, 3 →O

Chapter 1 - Conditionals

Dona Warren, Department of Philosophy, The University of Wisconsin – Stevens Point 30

The arrow between “A → C” and “B → D” is the main connector on line 1, so here’s a correct

proof of the argument:

1. (A → C ) → (B → D) P

2. A → C P

3. B P - want D

4. B → D 1, 2 →O

5. D 3, 4 →O

Can you see how this proof uses →O correctly? Each instance of →O is applied only to the main

connector of the conditional.

By the way, if you look at that proof and get all confused, you’re probably trying to take in the

whole thing at once. Follow it through one line at a time. When you look at line 4, see how it

follows from lines 1 and 2. Don’t bother with line 3 because line 3 isn’t used to get line 4. Then

when you look at line 5, see how it follows from lines 3 and 4. Don’t bother with line 2 because

line 2 isn’t used to get line 5. Never swallow a proof whole. Chew thoroughly. Like this:

1. (A → C ) →→→→ (B → D) P

2. A → C P

3. B P - want D

4. B → D 1, 2 →O (See how this line follows from 1 and 2?)

5. D 3, 4 →O

1. (A → C ) → (B → D) P

2. A → C P

3. B P - want D

4. B →→→→ D 1, 2 →O

5. D 3, 4 →O (See how this line follows from 3 and 4?)

Note: There can be more than one correct proof of an argument.

Consider, for example, the following two proofs of the same argument:

A → (B → C), D → B, A, D |- C

Proof 1 Proof 2

1. A → (B → C) P 1. A → (B → C) P

2. D → B P 2. D → B P

3. A P 3. A P

4. D P – want C 4. D P – want C

5. B → C 1, 3 →O 5. B 2, 4 →O

6. B 2, 4 →O 6. B → C 1, 3 →O

7. C 5, 6 →O 7. C 5, 6 →O

Chapter 1 - Conditionals

Dona Warren, Department of Philosophy, The University of Wisconsin – Stevens Point 31

Both of these proofs are correct; they simply draw the inferences in a different order. In Proof 1,

“B → C” is inferred before “B.” In Proof 2, “B” is inferred before “B → C.” Because both

proofs use →O correctly, however, both proofs are fine.

In general, as arguments become more complex, they will be open to a greater variety of proofs.

Different proofs might employ the same rules in a different order, as we’ve seen above, or

(especially as the set of rules gets richer) they might employ slightly different rules. All of these

different proofs will be correct, as long as they properly use the inference rules that they employ.

What this means in practical terms, of course, is that if your proof disagrees with mine, your

proof can still be fine – as long as you’ve correctly used the inference rules.

Note: The same line can be used in multiple inferences.

We don’t “use lines up” when we deploy them in an inference. For example, let’s walk through

the proof for this argument:

P → (Q → R), L → P, L, (L → P) → Q |- R

1. P → (Q → R) P

2. L → P P

3. L P

4. (L → P) → Q P - want R

To start the proof, we could either use →O on lines 2 and 3

1. P → (Q → R) P

2. L → P P

3. L P

4. (L → P) → Q P - want R

or we could use →O on lines 2 and 4.

1. P → (Q → R) P

2. L → P P

3. L P

4. (L → P) → Q P - want R

It doesn’t matter which we decide to do first. I’ll do →O on lines 2 and 3 just because that’s

just what I decided.

1. P → (Q → R) P

2. L → P P

3. L P

4. (L → P) → Q P - want R

5. P 2, 3 →O

Chapter 1 - Conditionals

Dona Warren, Department of Philosophy, The University of Wisconsin – Stevens Point 32

Now, I’ll do →O on lines 2 and 4.

1. P → (Q → R) P

2. L → P P

3. L P

4. (L → P) → Q P - want R

5. P 2, 3 →O

6. Q 2, 4 →O

Note that line 2 was used twice: one in the inference to “P” on line 5 and again in the inference

to “Q” on line 6. That’s just fine. We don’t use lines up.

At this point, we can use →O on lines 1 and 5 (which might be hard to see at first only because

lines 1 and 5 are so far away from each other in the proof).

1. P → (Q → R) P

2. L → P P

3. L P

4. (L → P) → Q P - want R

5. P 2, 3 →O

6. Q 3, 4 →O

7. Q → R 1, 5 →O

And then we can complete the proof by using →O on lines 6 and 7.

1. P → (Q → R) P

2. L → P P

3. L P

4. (L → P) → Q P - want R

5. P 2, 3 →O

6. Q 3, 4 →O

7. Q → R 1, 5 →O

8. R 6, 7 →O

(By the way, that was a reasonably complex proof so don’t worry if you feel like you couldn’t

have done it on your own. After all, you’ve only seen a handful of proofs at this point. If you

were able to follow along, step by step, as I did the proof, you’re doing just fine.)

Finally, the construction of proofs is very much like solving puzzles. Some proofs, like some

puzzles, are easy; some proofs, like some puzzles, are hard. But, just as there are often hints, or

rules of thumb, that we can use to solve puzzles more easily, there are hints, or rules of thumb,

that we can use to make our way from the premises to the conclusion more easily. Forward hints

tell us what to do given what we have. Backward hints tell us what to do given what we want.

Each inference rule has its own hints. Here are the hints for arrow out.

Chapter 1 - Conditionals

Dona Warren, Department of Philosophy, The University of Wisconsin – Stevens Point 33

Forward Hint for →O: If you have a line of the form ���� → �, see if you have or can get a line of

the form �. If so, use →�.

Backward Hint for →O: If you want �, see if you have or can get a line of the form ���� → �. If

so, see if you have or can get a line of the form ���� and use →O.

The backward hint is particularly helpful because it basically tells us to see if we can find what

we want sitting as a consequent of something that we already have. If we can, then we should

feel happy because we might be able to get what we want using →O.

For example, suppose we want to prove the following argument:

[(P → Q) → R] → T, T → (L → M), (P → Q) → R |- L → M

1. [(P → Q) → R] → T P

2. T → (L → M) P

3. (P → Q) → R P - want L → M

The first thing we might notice is that our conclusion is sitting as the consequent on line 2.

1. [(P → Q) → R] → T P

2. T → (L → M) P

3. (P → Q) → R P - want L → M

If we could get the antecedent of line 2, T, then we could infer L→M using →O.

1. [(P → Q) → R] → T P

2. T → (L → M) P

3. (P → Q) → R P - want L → M

And T is sitting as the consequent of the conditional on line 1.

1. [(P → Q) → R] → T P

2. T → (L → M) P

3. (P → Q) → R P - want L → M

So if we could get the antecedent of the conditional on line 1, we could conclude T using →O.

And guess what? We already have the antecedent of the conditional on line 1. It’s just the

statement on line 3.

1. [(P → Q) → R] → T P

2. T → (L → M) P

3. (P → Q) → R P - want L → M

Chapter 1 - Conditionals

Dona Warren, Department of Philosophy, The University of Wisconsin – Stevens Point 34

So there we have it! If we do →O on lines 1 and 3, we can infer T. Then we can do →O on line 2,

to give us our conclusion. The proof looks like this:

1. [(P → Q) → R] → T P

2. T → (L → M) P

3. (P → Q) → R P - want L → M

4. T 1, 3 →O

5. L → M 2,4 →O

Mental Proofs

One last word before presenting some practice arguments: Although constructing written proofs

is a handy skill and can be rather fun, in most contexts outside a logic classroom, it’s much more

useful to be able to construct a proof in your head, mentally tracing the inferences.

For example, let’s consider this argument: L→M, L, M→(A→B) |- A→B

Can you see that it’s valid by doing the proof in your head?

Obviously, I can’t write down exactly what I see in my own head, but roughly speaking, here’s

how it goes:

Of course, to be really useful, we need to be able to do this with arguments written in English. I

suppose that we should, ideally, be able to symbolize and prove mentally, but I don’t do that. I

find it much easier to actually write out the symbolization, especially if the sentences are written

in anything other than standard “if…then…” form, as they often are. I can then do the proof in

my head (unless, of course, the proof is very difficult).

For example, let’s consider this argument: “If all of the parties to the contract are competent then

the contract is binding, provided that all of the signers had an opportunity to read it. All of the

signers had an opportunity to read the contract. And all of the parties to the contract are

competent. Therefore the contract is binding.”

Can you sketch out a symbolization of this argument on paper and then prove it in your head?

This is roughly how I do it:

The argument is: L→M, L, M→(A→B) |- A→B.

Well, from L→M and L we get M. And from M

and M→(A→B) we get A→B.

So this argument is valid.

Chapter 1 - Conditionals

Dona Warren, Department of Philosophy, The University of Wisconsin – Stevens Point 35

Because in order for symbolic logic to be as useful to us as possible we need to be able to do

some proofs in our head, I’ll be asking you to try to do some proofs mentally. Don’t worry at all

if this is difficult for you at first. If you can’t do a proof mentally, do a written proof for the

argument, leave the argument for five minutes or so, and then come back and see if you can do

the proof again, this time entirely in your head. With practice, it gets much easier.

“If all of the parties to the contract are competent then

the contract is binding, provided that all of the signers

had an opportunity to read it. All of the signers had an

opportunity to read the contract. And all of the parties

to the contract are competent. Therefore the contract is

binding.”

Okay, that’s a bit confusing. I’d better symbolize it.

If all of the parties to the contract are competent then the contract is binding,

provided that all of the signers had an opportunity to read it.

C → (R → B)

All of the signers had an opportunity to read the contract.

R

And all of the parties to the contract are competent.

C

Therefore the contract is binding.”

B

That’s better! Looking that this symbolization, I see

that C → (R → B) and the C gives us R →B. And we

already have R, so that gives us B.

The argument is valid.

Chapter 1 - Conditionals

Dona Warren, Department of Philosophy, The University of Wisconsin – Stevens Point 36

Practice

Prove the following arguments valid. The answers follow in the endnotes.

1) A →B, A, B →C |- C92

2) R → (T →S), R, T |- S93

3) M →N, (M →N) →Q, Q →T |- T94

4) P →Q, (P →Q) → (A →F), A |- F95

5) C, D→(F→H), (A→B)→(C→D), A→B |- F→H96

6) Do this proof mentally: R→S, R, S → (A → B) |- A → B97

7) Do this proof mentally: R→S, R, S → (A → B), A |- B98

8) Do this proof mentally: R→S, R, S → (A → B), A, B→C |- C99

9) B→(L→R), [B→(L→R)]→B |- L→R100

10) A→B, A, (A→B)→(B→D) |- D101

11) (W → B) → [C → (A →W)], C, A, W → B |-B102

12) P→Q, {B→[(P→Q)→R]}→(A→B), A, B→[(P→Q)→ R] |- R103

13) Do this proof mentally: “If God exists then miracles are possible. And God does exist.

Therefore miracles are possible.”104

14) Do this proof mentally: “Intelligent origin is necessary for an ordered universe. If the

universe has intelligent origin then God exists. The universe is ordered. Therefore, God

exists.”105

15) Do this proof mentally: “People have an innate sense of right and wrong. People can have an

innate sense of right and wrong only if God exists. And provided that God exists, evil-doers will

be punished. Therefore, evil-doers will be punished.”106

16) Do this proof mentally: “If utilitarianism is right then lying can be morally permissible if it

sometimes serves to promote the greatest happiness for the greatest number of people. In fact,

utilitarianism is right. And lying does sometimes serve to promote the greatest happiness for the

greatest number of people. Therefore, lying can be morally permissible.”107

Chapter 1 - Conditionals

Dona Warren, Department of Philosophy, The University of Wisconsin – Stevens Point 37

17) “If Kant is right then ethical acts must pass the categorical imperative. Kant is right. Lying is

ethically wrong if lying is unable to become the universal law, provided that ethical acts must

pass the categorical imperative. If lying would make communication and society impossible then

it’s unable to become a universal law. Lying would make communication and society impossible.

Therefore lying is ethically wrong.”108

ESTABLISHING INVALIDITY

If we can construct a proof for an argument then we know that the argument is valid. However, if

we can’t construct a proof for an argument, we don’t thereby know that the argument is invalid.

This is because there are lots of reasons why we might not be able to construct a proof for an

argument. Maybe the argument is invalid, but maybe the argument is valid and we can’t prove it

because we’re tired, or because we haven’t learned all of the rules we need yet.

So, how can we show that an argument is invalid? We’ll see two ways.

• The Truth Table Test

First, we can exploit the fact that in a valid argument, whenever the premises are true the

conclusion must be true as well. In other words, in a valid argument, the premises can’t be true

and the conclusion false at the same time. To show that an argument is invalid, then, all we need

to do is demonstrate that the premises can be true and the conclusion false at the same time, and

in order to do that we’ll assign truth values to sentence letters.v

This is sometimes called “the Truth Table Test” and we can summarize it as follows:

The Truth Table Test for Validity and Invalidity:

An argument is valid. = It’s impossible for the premises to be true and the conclusion to be false.

An argument is invalid. = It’s possible for the premises to be true and the conclusion to be false.

-So-

If we can’t make the premises true and the conclusion false then the argument is valid.

If we can make the premises true and the conclusion false then the argument is invalid.

Get it? A few of examples will probably make this a lot clearer.

v Since we can show that an argument is invalid by demonstrating that the premises can all be true and the

conclusion false at the same time, can we show that an argument is valid by demonstrating that the premises can all

be true and the conclusion true at the same time? No. If we know that an argument has true premises and a true

conclusion, we don’t know that the argument is valid because it may have simply gotten lucky (e.g. “Many women

are over 5 feet tall. Therefore Madison is the capitol of Wisconsin.”) However, if we know that an argument has true

premises and a false conclusion then we know that the argument must be invalid because we know that the truth of

the premises doesn’t “force” the truth of the conclusion.

The key question isn’t, “If this argument has true premises, can it have a true conclusion?” The key

question is, “If this argument has true premises, must it have a true conclusion?”

Chapter 1 - Conditionals

Dona Warren, Department of Philosophy, The University of Wisconsin – Stevens Point 38

Let’s start by considering the valid argument form we’ve been studying:

P → Q, P |- Q

And let’s see if we can make the premises true and the conclusion false at the same time.

Well, if the premises are true, then P must be true because P is the second premise.

P → Q, P |- Q

t T

And if the conclusion is false then Q must be false because Q is the conclusion.

P → Q, P |- Q

t f T F

But now, what about the first premise? If P is true and Q is false, then P→Q is false.

P → Q, P |- Q

t F f T F

We’ve seen that we can’t make the premises true and the conclusion false at the same time. This

means that whenever the premises are true, the conclusion must be true as well. And this, in turn,

means that the argument is valid (which is hopefully what you expected all along).

Now let’s turn to another argument:

P → Q, Q → R |- P → R

Once again, we’ll see if we can make the premises true and the conclusion false at the same time.

There are a number of ways that the premises can be true, but there’s only one way for the

conclusion to be false: P must be true and Q must be false.

P → Q, Q → R |- P → R

t f t F f

Now, if the premise P→Q is to be true, Q has to be true.

P → Q, Q → R |- P → R

t T t t f t F f

But this makes the second premise, Q → R, false.

P → Q, Q → R |- P → R

t T t t F f t F f

Chapter 1 - Conditionals

Dona Warren, Department of Philosophy, The University of Wisconsin – Stevens Point 39

Once again, we’ve seen that we can’t make the premises true and the conclusion false and so this

argument is valid even though we can’t construct a proof for this argument yet. (We will be able

to construct a proof for this argument later on.vi

)

Finally, let’s look at this argument:

P → Q, Q |- P

Can the premises be true and the conclusion false? Well, if the premises are true then Q must be

true because Q is a premise.

P → Q, Q |- P

t T

And of course, P must be false because that’s the conclusion.

P → Q, Q |- P

f t T F

But now, what about the premise P→Q? That premise is true! (Remember that a conditional is

true if the antecedent is false and the consequent is true.)

P → Q, Q |- P

f T t T F

This shows that the premises can be true and the conclusion false at the same time.

In other words, the truth of the premises doesn’t force the truth of the conclusion.

This argument, then, is invalid.

• Spotting Formal Fallacies

The invalid argument form that we’ve just seen is very important because it shows up a lot and

can fool the unwary. After all, it’s so temptingly close to Arrow Out, a valid argument form!

→O - Valid Invalid

P → Q

P

Therefore Q.

P → Q

Q

Therefore P.

To see the difference between these argument forms more clearly, it helps to use the “Poodle /

Dog” example.

vi Since we can use this method to establish validity, why bother with proofs? We’ll continue to focus extensively

upon proofs because a proof corresponds – at least roughly - to the thought process behind an argument and because

we want to use logic to think through arguments. (Besides, in predicate logic truth tables won’t work anymore, but

proofs still will.)

Chapter 1 - Conditionals

Dona Warren, Department of Philosophy, The University of Wisconsin – Stevens Point 40

→O - Valid Invalid

P → Q

P

Therefore Q.

P → Q

Q

Therefore P.

If Max is a poodle then Max is a dog.

Max is a poodle.

Therefore Max is a dog.

If Max is a poodle then Max is a dog.

Max is a dog.

Therefore Max is a poodle.

The first argument works because, given the fact that all poodles are dog (premise 1), if we’re

told that Max is a poodle (premise 2), we can conclude that Max is a dog.

The second argument fails because, given the fact that all poodles are dog (premise 1), if we’re

told that Max is a dog (premise 2), we can’t conclude that Max is a poodle. Max could be a

terrier, or a schnauzer, or a Chihuahua.

Here’s another way to think about this that many people find helpful.

Consider the following map:

Now, suppose that we run into a friend in Q-City. Can we conclude that he took Highway P to

get there? No, because he could have taken Highway R or Highway S instead.

In other words, although it’s true that if our friend took Highway P then he’d be in Q-City

(i.e. “P → Q” is true), and although it’s true that our friend is in Q-City (i.e. “Q” is true), it does

not follow that our friend took Highway P (i.e. “P” doesn’t follow).

“P→Q, Q |- P” is invalid.

This map shows us that there are

at least three ways to reach Q-City.

If we take Highway P then we’ll

get to Q-City, so “P → Q” is true.

If we take Highway R then we’ll

get to Q-City, so “R → Q” is true.

And if we take Highway S then

we’ll get to Q-City, so “S → Q” is

true.

Q-CITY

HIGHWAY P

HIGHWAY S

HIGHWAY R

Hi!

Chapter 1 - Conditionals

Dona Warren, Department of Philosophy, The University of Wisconsin – Stevens Point 41

Since this kind of faulty inference is so very common, it’s worthwhile to take it as a formal

fallacy. Formal fallacies are argument forms that specify common but incorrect inferences and

they’re very useful to know. This formal fallacy is called in the “Fallacy of Assuming the

Consequent,” because it involves assuming that the consequent of a conditional is true and

inferring that the antecedent of the conditional is true.

→O - Valid Fallacy of Assuming the Consequent - Invalid

P → Q

P

Therefore Q.

P → Q

Q

Therefore P.

If Max is a poodle then Max is a dog.

Max is a poodle.

Therefore Max is a dog.

If Max is a poodle then Max is a dog.

Max is a dog.

Therefore Max is a poodle.

FAC (Fallacy of Assuming the Consequent): ���� → �, � |- ����.

No! Don’t do this! It doesn’t work! This is an invalid inference! Beware!

It’s good to be familiar with formal fallacies because it’s much more common to recognize that

an argument is invalid by seeing that it commits a formal fallacy than it is to recognize that an

argument is invalid by employing the truth table method. In fact, the only time I use truth tables

is when I’m teaching formal logic; in “real life,” I look for formal fallacies instead. If every

proof for an argument depends upon the commission of a formal fallacy then I know that the

argument is invalidvii

.

For example, consider the argument: A, A → (R → S), S | - R. Is this argument valid or invalid?

Can we prove it using only valid inferences, or does any proof depend upon the commission of a

fallacy? Let’s see:

1. A P

2. A → (R → S) P

3. S P – want R

4. R → S 1,2 MP

5. R 3,4 FAC!

Because this proof compels us to commit the Fallacy of Assuming the Consequent, this argument

is invalid. The conclusion does not logically follow from the premises.

vii

Technically speaking, any “proof” that commits a formal fallacy isn’t a real proof because a real proof contains

only valid inferences. But I’ll speak in this looser way because it’s less cumbersome and more natural and because I

don’t think that this relaxed use of the term will cause any serious misunderstandings.

Chapter 1 - Conditionals

Dona Warren, Department of Philosophy, The University of Wisconsin – Stevens Point 42

Mental Evaluation

As with recognizing that arguments are valid by constructing proofs for them, the ability to

recognize that an argument is valid by detecting that it commits a formal fallacy is the most

useful when we can carry it out mentally.

For example, let’s consider this argument: L→M, L, (A→B)→M |- A→B

Can you see that it’s invalid by attempting to do the proof in your head?

Here’s roughly how I think about it:

No let’s consider this English argument: “If all of the parties to the contract are competent, then

the contract is binding only if all of the signers had an opportunity to read it. All of the signers

had an opportunity to read the contract. And all of the parties to the contract are competent.

Therefore the contract is binding.”

Can you sketch out a symbolization of this argument on paper and then try to prove it in your

head?

This is roughly how I do it:

The argument is: L→M, L, (A→B)→M |- A→B.

Well, from L→M and L we validly get M. But in order

to go from M and (A→B)→M to A→B, we’d need to

commit the Fallacy of Assuming the Consequent.

So this argument is invalid.

“If all of the parties to the contract are competent, then

the contract is binding only if all of the signers had an

opportunity to read it. All of the signers had an

opportunity to read the contract. And all of the parties

to the contract are competent. Therefore the contract is

binding.”

That’s a bit confusing. I’d better symbolize it.

Chapter 1 - Conditionals

Dona Warren, Department of Philosophy, The University of Wisconsin – Stevens Point 43

In order to help you learn how to assess validity in your head, I’ll be asking you to try to do some

evaluations mentally. Remember, don’t worry at all if this is difficult for you at first. If you can’t

evaluate an argument mentally, attempt a written proof first. Then leave the argument for five

minutes or so, come back to it again, and see if you can evaluate it entirely in your head. With

practice, it gets much easier.

Practice

Determine whether the arguments are valid or invalid by attempting to prove them. The answers

follow in the endnotes.

1. (P → Q) → (R → S), R, P→Q, |- S109

2. (P → Q) → (R → S), S, P→Q, |- R110

3. R, B→A, A→C, C→(R→S), B |- S111

4. L → (P → Q), S→R, P, L, Q→R |- S112

If all of the parties to the contract are competent, then the contract is binding

only if all of the signers had an opportunity to read it.

C → (B → R)

All of the signers had an opportunity to read the contract.

R

And all of the parties to the contract are competent.

C

Therefore the contract is binding.”

B

That’s better! Looking that this symbolization, I see

that C → (B → R) and the C gives us B →R. But in

order to go from R and B→R to B, we’d need to

commit the Fallacy of Assuming the Consequent.

This argument is invalid.

Chapter 1 - Conditionals

Dona Warren, Department of Philosophy, The University of Wisconsin – Stevens Point 44

5. Assess mentally: L→M, (L→M) → (A→B), B | A113

6. Assess mentally: L→M, (L→M) → (A→B), A | B114

7. Write the symbolization and assess mentally: “If God exists then the world would be well-

designed. And the world is well designed. Therefore God exists.”115

8. Write the symbolization and assess mentally: “If the world is well-designed then God exists.

And the world is well designed. Therefore God exists.”116

9. Write the symbolization and assess mentally: “People have an innate sense of right and wrong.

People can have an innate sense of right and wrong only if God exists. And provided that God

exists, evil-doers will be punished. Therefore, evil-doers will be punished.”117

10. Write the symbolization and assess mentally: “People have an innate sense of right and

wrong. People can have an innate sense of right and wrong if God exists. And provided that God

exists, evil-doers will be punished. Therefore, evil-doers will be punished.” 118

11. “If lying can be morally permissible provided that it sometimes serves to promote the

greatest happiness for the greatest number of people then utilitarianism is right. In fact,

utilitarianism is right. And lying does sometimes serve to promote the greatest happiness for the

greatest number of people. Therefore, lying can be morally permissible.”119

12. “If utilitarianism is right then lying can be morally permissible provided that it sometimes

serves to promote the greatest happiness for the greatest number of people. In fact, utilitarianism

is right. And lying does sometimes serve to promote the greatest happiness for the greatest

number of people. Therefore, lying can be morally permissible.”120

WHERE NEXT?

Where next? Well, we’ve seen one valid inference form, Arrow Out, which tells us that from

P →Q, and P, we may infer Q. Arrow Out has another name, modus ponens, which is Latin for

“the way of affirming.” (This name makes sense because Arrow Out involves affirming the

antecedent of a conditional and then affirming the consequent of the conditional.)

At this point, I’m tempted to introduce the inference rule modus tollens, which tells us that from

P→Q, and Not-Q, we may infer Not-P. The name “modus tollens” is Latin for “the way of

denying,” (This name makes sense because modus tollens involves denying the consequent of a

conditional and then denying the antecedent of the conditional.)

It would be nice to discus modus tollens immediately after modus ponens because modus tollens

and modus ponens form a “matched set,” so to speak.

Chapter 1 - Conditionals

Dona Warren, Department of Philosophy, The University of Wisconsin – Stevens Point 45

Modus Ponens Modus Tollens

P → Q, P |- Q P → Q, Not-Q |- Not-P

There is, however, an excellent reason to not introduce modus tollens right now: unlike modus

ponens, which is obviously valid, modus tollens can really puzzle people. (If you find modus

tollens to be just as obvious as modus ponens, good for you. You’re in the minority.) Because

people can be unsure that modus tollens is valid, we won’t want to take it as a primitive inference

rule; we’ll want to derive it. But the best way to derive modus tollens is to use a method called

“proof by contradiction,” and in order to carry out a proof by contradictions, we need to be

familiar with conjunctions, conditional proofs, and negations. Our next order of business, then,

will be to learn about these topics, and about all of the other incredibly useful things associated

with them.

Chapter 1 - Conditionals

Dona Warren, Department of Philosophy, The University of Wisconsin – Stevens Point 46

CHAPTER SUMMARY

UNDERSTANDING COMPLEX SENTENCES If ���� then ���� = ���� →→→→ ����

• ���� → ���� is false if and only if ���� can be true and ���� can be false at the same time. • Whatever follows “if” (“should” and “provided that”) becomes the antecedent. • Whatever follows “only if” becomes the consequent. • Whatever is being described as sufficient becomes the antecedent. • Whatever is being described as necessary becomes the consequent.

If the sentence has more than one connector, we identify the main connector by

i) Identifying all of the connectors.

ii) Seeing what each connector connects.

iii) If there is nothing left over, that connector may be the main connector. If there is part of the

sentence left over, that connector isn’t the main connector. Once we’ve identified the main connector, we symbolize “around” that main connector To see if two sentences say the same thing, symbolize the sentences and see if the symbolizations are the same or equivalent to each other (see the equivalence rules below). If so, then the sentences say the same thing. If not, then they don’t. ASSESSING ARGUMENTS FOR VALIDITY / INVALIDITY If we can proceed from the premises of an argument to its conclusion by a series of valid inference rules then the argument as a whole is valid. If we can proceed from the premises of an argument to its conclusion only by committing a formal fallacy then the argument as a whole is invalid. Valid Inference Rules → O ����→→→→����, ���� |- ���� Formal Fallacies

FAC ���� →→→→ ����, ���� |- ����

Chapter 1 - Conditionals

Dona Warren, Department of Philosophy, The University of Wisconsin – Stevens Point 47

ANSWERS

Symbolizing Conditionals with One Connector

I) Symbolizing with “If…. then….”

Figuring Things Out

1 1. “If you’re a physics major then your major is physics.”

P→P

We use “P” for the antecedent and the consequent because “You’re a physics major” and

“Your major is physics” is the same idea.

2 2. “If you’re taking a physics class then you’re taking a science class.”

P→S

3 P→S is true.

Physics classes are science classes so if you’re taking physics, you’re taking science.

4 3. “If you’re taking a science class then you’re taking a physics class.”

S→P

5 S→P isn’t true.

You could be taking a biology or chemistry class instead.

6 ���� → � ≠ � → ����.

This is an important logic fact. Order matters when we have conditionals.

II) Symbolizing with “If,” “Should,” and “Provided that”

Figuring Things Out

7 1. “If Jim is a pediatrician, he’s a doctor.”

P → D

8 2. “Jim is a doctor, if he’s a pediatrician.”

P → D

9 Rule: If ����, � = � → � .

Rule: ����, if � = � → ���� .

Rule: In general, whatever follows “if” becomes the antecedent of the conditional.

10

3. “Should the movie be good, we’ll recommend it.”

Chapter 1 - Conditionals

Dona Warren, Department of Philosophy, The University of Wisconsin – Stevens Point 48

M → R

11

4. We’ll recommend the movie, should it be good.”

M → R

12

5. “Provided that it’s a nice day, we’ll go on a walk.”

N → W

13

6. “We’ll go on a walk, provided that it’s a nice day.”

N → W

14

Rule: “Should” and “provided that” work exactly like “if” and whatever follows them

becomes the antecedent of the conditional.

Practice

15

1. “If C, D.”

C → D

16

2. “A, if B.”

B → A

17

3. “P, provided that Q.”

Q → P

18

4. “Provided that R, S.”

R → S

19

5. “Should L, A.”

L → A

20

6. “A, should P.”

P → A

21

1. “If the painting is expensive then he’ll like it.”

“If the paining is expensive, he’ll like it.”

“He’ll like the paining, if it’s expensive.”

22

2. “If the class is open then we’ll enroll.”

“Should the class be open, we’ll enroll.”

“We’ll enroll, should the class be open.”

23

3. “If computers can think then minds are physical.”

“Provided that computers can think, minds are physical.”

“Minds are physical, provided that computers can think.”

Chapter 1 - Conditionals

Dona Warren, Department of Philosophy, The University of Wisconsin – Stevens Point 49

III) Symbolizing with “Sufficient” and “Necessary”

Figuring Things Out

24

1. “Getting 100% is a sufficient condition for passing the test.”

H → P

25

2. “Getting 100% is sufficient for passing the test.”

H → P

26

3. “To pass the test, it’s sufficient to get 100%.”

H → P

27

Rule: Whatever is being described as sufficient becomes the antecedent of the

conditional.

28

4. “Completing the foreign language requirement is a necessary condition for obtaining your

degree.”

D → F

29

5. “Completing the foreign language requirement is necessary for obtaining your degree.”

D → F

30

6. “To obtain your degree, it’s necessary to complete the foreign language requirement.”

D → F

31

Rule: Whatever is being described as necessary becomes the consequent of the conditional.

Practice

32

1. “Q is sufficient for D.”

Q → D

33

2. “For E, it’s sufficient that O.”

O → E

34

3. “L is a sufficient condition for Q.”

L → Q

35

4. “A is necessary for B.”

B → A

36

5. “R is a necessary condition for C.”

C → R

Chapter 1 - Conditionals

Dona Warren, Department of Philosophy, The University of Wisconsin – Stevens Point 50

37

6. “For T, it’s necessary that D.”

T → D

38

1. “If Robin is a surgeon then she’s smart.”

“Robin being a surgeon is sufficient for her being smart.”

“Robin being smart is necessary for her being a surgeon.”

39

2. “If the power was out then the clocks will be blinking.”

“The power being out is a sufficient condition for the clocks blinking.”

“The clocks blinking is a necessary condition for the power having been out.”

40

3. “If ghosts exist then minds are nonphysical.”

“Minds being nonphysical is necessary for ghosts existing.”

“Ghosts existing is sufficient for minds being nonphysical.”

IV) Symbolizing with “Only if”

41

Rule: ���� only if � = ���� → � .

Rule: In general, whatever follows “only if” becomes the consequent of the conditional.

Practice

42

1. “Democracy will work only if the citizens are well-informed.”

D → W

43

2. “Only if the citizens are well-informed, can democracy work.”

D → W

44

3. “You can join the club only if you pay the dues.”

J → P

45

4. “Only if you play the dues, can you join the club.”

J → P

46

5. “P only if Z.”

P → Z

47

6. “P, if Z.”

Z → P

48

7. “If L, M.”

L → M

49

8. “Only if L, M.”

Chapter 1 - Conditionals

Dona Warren, Department of Philosophy, The University of Wisconsin – Stevens Point 51

M → L

50

1. “If Jill enjoyed the movie then it must be a foreign film.”

“Jill enjoyed the movie only if it’s a foreign film.”

“It must be a foreign film, if Jill enjoyed it.”

51

2. “If the cookie is good for you then it contains raisins.”

“The cookie contains raisins, if it’s good for you.”

“The cookie is good for you only if it contains raisins.”

52

3. “If Hank won then the election was rigged.”

“Hank won only if the election was rigged.”

“The election was rigged, if Hank won.”

V) Learning how to Symbolize: “Meaning” vs. “Rules”

Practice

53

1. “If behaviorism is right then mental states are dispositions to behave.”

B → D

54

2. “Behaviorism is right only if mental states are dispositions to behave.”

B → D

55

3. “For behaviorism to be right, it’s sufficient that mental states be dispositions to behave.”

D → B

56

4. “Should behaviorism be right, mental states are dispositions to behave.”

B → D

57

5. “Behaviorism is right, provided that mental states are dispositions to behave.”

D → B

58

6. “Mental states being dispositions to behave is a necessary condition for behaviorism being

right.”

B → D

59

7. “Behaviorism is right, if mental states are dispositions to behave.”

D → B

60

8. “Only if mental states are dispositions to behave, is behaviorism right.”

B → D

61

9. “Behaviorism is right, should mental states be dispositions to behave.”

D → B

Chapter 1 - Conditionals

Dona Warren, Department of Philosophy, The University of Wisconsin – Stevens Point 52

62

1 0. “For mental states to be dispositions to behave, it’s necessary that behaviorism is right.”

D → B

Recognizing Equivalent Conditionals

Practice

Are the following sentences equivalent to

“If our actions have moral value then we have free will.”?

63

1. “Our actions have moral value only if we have free will.”Equivalent

64

2. “Our actions have moral value, if we have free will.” Not equivalent

65

3. “Our actions have moral value, provided that we have free will.” Not equivalent

66

4. “Our actions having moral value is a sufficient condition for our having free will.”

Equivalent

67

5. “Our having free will is a necessary condition for our actions having moral value.”

Equivalent

Are the following sentences equivalent to

“Having a shape is a necessary condition for having a color.”?

68

1. “Something has a shape only if it has a color.” Not equivalent

69

2. “Something has a color only if it has a shape.” Equivalent

70

3. “Something has a shape if it has a color.” Equivalent

71

4. “Having a color is a sufficient condition for having a shape.” Equivalent

72

5. “Something has a color, provided that it has a shape.” Not equivalent

Assessing Conditionals

I) What Makes a Conditional False

73

Rule: A conditional is false when and only when the antecedent is true and the consequent is

false.

II) Assessing Conditionals

Chapter 1 - Conditionals

Dona Warren, Department of Philosophy, The University of Wisconsin – Stevens Point 53

Practice

For each of the claims below identify the objection that, if true, would show that claim to be false.

74

1. “If we can’t know anything for certain then knowledge isn’t worth seeking.”

a. Although we can’t know anything for certain, knowledge is worth seeking.

b. Although we can know some things for certain, knowledge isn’t worth seeking.

75

2. “If a belief is useful then we’re justified in holding it.”

a. We’re justified in holding some beliefs even though they’re not useful.

b. We’re not justified in holding some beliefs even though they are useful.

76

3. “If euthanasia maximizes happiness then it’s ethical.”

a. Some things that maximize happiness aren’t ethical.

b. Some things that are ethical don’t maximize happiness.

77

4. “Lying is unethical only if it violates our culture’s moral code.”77

U → V

a. Lying is unethical even if it doesn’t violate our culture’s moral code.

b. Lying violates our culture’s moral code even if it isn’t unethical.

78

5. “Provided that some beliefs are justified without argument, religious faith is philosophically

acceptable.”

a. Even though no beliefs are justified without argument, religious faith is philosophically

acceptable.

b. Even though some beliefs are justified without argument, religious faith isn’t

philosophically acceptable.

79

6. “God doesn’t know our future actions, if we have free will.” Free will → God doesn’t know

a. “God knows our future actions but we have free will anyway.”

b. “God doesn’t know our future actions but we don’t have free will anyway.”

80

7. “We have good reason to believe in God, provided that we have good reason to believe in

miracles.” M → G

a. Even though we don’t have good reason to believe in God, we do have good reason to

believe in miracles.

b. Even though we have good reason to believe in God, we don’t have good reason to believe

in miracles.

81

8. “Cross-cultural agreement about ethics is a necessary condition for the objectivity of

morality.” O → A

a. Although morality is objective, there can still fail to be cross-cultural agreement about

ethics.

b. Even if there is cross-cultural agreement about ethics, morality can still fail to be objective.

Chapter 1 - Conditionals

Dona Warren, Department of Philosophy, The University of Wisconsin – Stevens Point 54

82

9. “For an action to be moral it’s sufficient that it satisfy Kant’s Categorical Imperative.”

K → M

a. An action can be moral without satisfying Kant’s Categorical Imperative.

b. An action can satisfy Kant’s Categorical Imperative without being moral.

Symbolizing Conditionals with Multiple Connectors

Practice

83

1. “If it’s the case that computers deserve moral consideration if they can think, then the ability

to think is the source of our moral value.”

C = Computers can think

S = The ability to think is the source of our moral value.

M = Computers deserve moral consideration.

(C → M) → S

84

2. “If computers can think then provided that the ability to think is the source of our moral

value, computers deserve moral consideration.”

C = Computers can think

S = The ability to think is the source of our moral value.

M = Computers deserve moral consideration.

C → (S → M)

85

3. “Provided that being able to carry on a conversation is sufficient for being able to think, the

Turing Test is an accurate indicator of artificial intelligence.”

C = x can carry on an intelligent conversation.

T = x can think.

A = The Turing Test is an accurate indicator of artificial intelligence.

(C → T) → A

86

4. “Being able to carry on an intelligent conversation is necessary for having a mind only if

many people don’t have minds.”

C = x can carry on an intelligent conversation.

M = x has a mind.

D = Many people don’t have minds.

(M → C) → D

Chapter 1 - Conditionals

Dona Warren, Department of Philosophy, The University of Wisconsin – Stevens Point 55

87

5. “If our being able to think is sufficient for our having moral value, then if computers can

think, they deserve moral consideration.”

T = We can think.

V = We have moral value.

C = Computers can think.

M = Computers deserve moral consideration.

(T → V) → (C → M)

88

6. “Being able to think is necessary for having moral value only if being able to think is

sufficient for having a soul.”

T = x can think.

V = x has moral value.

S = x has a soul.

(V → T) → (T → S)

89

7. “Should being able to carry on an intelligent conversation be sufficient for having a mind,

then an answering machine would have a mind if it could answer back.”

C = x can carry on an intelligent conversation.

M = x has a mind.

A = An answering machine has a mind.

B = An answering machine can answer back.

(C → M) → (B → A)

90

8. “If a computer could carry on an intelligent conversation then provided that being able to

carry on an intelligent conversation is sufficient for having a mind, the computer would have

a mind.”

C = A computer can carry on an intelligent conversation.

M = x has a mind.

I = x can carry on an intelligent conversation

H = A computer has a mind.

C → ((I → M) → H )

91

9. “Neither side can avoid circularity. This is disturbing if a non-question-begging way of

mutually resolving basic disagreements over the overall force of a body of evidence is a

necessary condition of the possibility of being rational in these matters.” (William J.

Wainwright, Reason and the Heart, Cornel University Press, 1995, p. 123)

Chapter 1 - Conditionals

Dona Warren, Department of Philosophy, The University of Wisconsin – Stevens Point 56

Symbolize the second sentences in this quotation, using the following sentence letters:

D = It’s disturbing that neither side can avoid circularity.

N = There is a non-question-begging way of mutually resolving basic disagreements over the

overall force of a body of evidence.

R = One can be rational in matters involving basic disagreements over the overall force of a

body of evidence

(R → N) → D

INFERENCE RULE: → O

ESTABLISHING VALIDITY: FORMAL PROOFS

Practice

92

1. A →B, A, B →C |- C

1. A →B P

2. A P

3. B →C P - want C

4. B 1, 2 →O

5. C 3, 4 →O

93

2. R → (T →S), R, T |- S

1. R → (T →S) P

2. R P

3. T P- want S

4. T →S 1, 2 →O

5. S 3, 4 →O

94

3. M →N, (M →N) →Q, Q →T |- T

1. M →N P

2. (M →N) →Q P

3. Q →T P - want T

4. Q 1, 2 →O

5. T 3, 4 →O

95

4. P →Q, (P →Q) → (A →F), A |- F

1. P →Q P

Chapter 1 - Conditionals

Dona Warren, Department of Philosophy, The University of Wisconsin – Stevens Point 57

2. (P →Q) → (A →F) P

3. A P - want F

4. A →F 1, 2 →O

5. F 3, 4 →O

96

5. C, D→(F→H), (A→B)→(C→D), A→B |- F→H

1. C P

2. D→(F→H) P

3. (A→B)→(C→D) P

4. A→B P - want F→H

5. C → D 3,4 →O

6. D 1,5 →O

7. F→H 2,6 →O

97

6. Do this proof mentally: R→S, R, S → (A → B) |- A → B

R→S and R gives us S. And S and S→(A→B) gives us A→B.

98

7.Do this proof mentally: R→S, R, S → (A → B), A |- B

R→S and R gives us S. S and S→(A→B) gives us A→B. And A→B and A gives us B.

99

8. Do this proof mentally: R→S, R, S → (A → B), A, B→C |- C

R→S and R gives us S. S and S→(A→B) gives us A→B. A→B and A gives us B. And B

and B→C gives us C.

100

9. B→(L→R), [B→(L→R)]→B |- L→R

1. B→(L→R) P

2. [B→(L→R)]→B P – want L→R

3. B 1,2 →O

4. L →R 2,3 →O

Note: Line 2 was used twice. That’s okay. We never use lines up.

101

10. A→B, A, (A→B)→(B→D) |- D

1. A→B P

2. A P

3. (A→B)→(B→D) P - want D

4. B 1,2 →O

Chapter 1 - Conditionals

Dona Warren, Department of Philosophy, The University of Wisconsin – Stevens Point 58

5. B→D 1,3, →O

6. D 4,5 →O

102

11. (W → B) → [C → (A →W)], C, A, W → B |-B

1. (W →B) → [C → (A →W)] P

2. C P

3. A P

4. W →B P – want B

5. C → (A →W) 1, 4 →O

6. A →W 2, 5 →O

7. W 3, 6 →O

8. B 4, 7 →O

103

12. P→Q, {B→[(P→Q)→R]}→(A→B), A, B→[(P→Q)→ R] |- R

1. P→Q P

2. {B→[(P→Q)→R]}→(A→B) P

3. A P

4. B→[(P→Q)→ R] P - want R

5. A→B 2, 4 →O

6. B 3, 5 →O

7. (P→Q) → R 4, 6 →O

8. R 1, 7 →O

104

13. Do this proof mentally: “If God exists then miracles are possible. And God does exist.

Therefore miracles are possible.”

G → M, G |- M

From G→M and G, M follows directly.

105

14. Do this proof mentally: “Intelligent origin is necessary for an ordered universe. If the

universe has intelligent origin then God exists. The universe is ordered. Therefore, God

exists.”

O → I, I → G, O |- G

From O→I and O, we get I. Then from I and I →G, we get G.

Written out, the proof looks like this:

1. O → I A

2. I → G A

Chapter 1 - Conditionals

Dona Warren, Department of Philosophy, The University of Wisconsin – Stevens Point 59

3. O A - want G

4. I 1, 3 →O

5. G 2, 4 →O

106

15. Do this proof mentally: “People have an innate sense of right and wrong. People can have

an innate sense of right and wrong only if God exists. And provided that God exists, evil-

doers will be punished. Therefore, evil-doers will be punished.”

I, I→G, G→P |- P

From I and I→G we get G. Then from G and G→P, we get P

Written out, the proof looks like this:

1. I A

2. I→G A

3. G→P A – want P

4. G 1,2 →O

5. P 3,4 →O

107

16. Do this proof mentally: “If utilitarianism is right then lying can be morally permissible if

it sometimes serves to promote the greatest happiness for the greatest number of people. In

fact, utilitarianism is right. And lying does sometimes serve to promote the greatest

happiness for the greatest number of people. Therefore, lying can be morally permissible.”

U → (G → L), U, G |- L

From U→(G→L) and U we get G→L. Then from G and G→L, we get L.

Written out, the proof looks like this:

1. U → (G → L) P

2. U P

3. G P - want L

4. G → L 1,2 →O

5. L 3,4 →O

108

17. “If Kant is right then ethical acts must pass the categorical imperative. Kant is right. Lying

is ethically wrong if lying is unable to become the universal law, provided that ethical acts

must pass the categorical imperative. If lying would make communication and society

impossible then it’s unable to become a universal law. Lying would make communication

and society impossible. Therefore lying is ethically wrong.”

K → C, K, C → (U → W), I → U, I |- W

Chapter 1 - Conditionals

Dona Warren, Department of Philosophy, The University of Wisconsin – Stevens Point 60

1. K → C P

2. K P

3. C → (U → W) P

4. I → U P

5. I P – want W

5. C 1,2 →O

6. U → W 3,5 →O

7. U 4,5 →O

8. W 6,8 →O

ESTABLISHING INVALIDITY

109

1. (P → Q) → (R → S), R, P→Q, |- S

1. (P → Q) → (R → S) P

2. R P

3. P→Q P - want S

4. R→S 1,3 →O

5. S 2,3 →O

Valid

110

2. (P → Q) → (R → S), S, P→Q, |- R

1. (P → Q) → (R → S) P

2. S P

3. P→Q P - want R

4. R→S 1,3 →O

5. R 2,3 FAC!

Invalid

111

3. R, B→A, A→C, C→(R→S), B |- S

1. R P

2. B→A P

3. A→C P

4. C→(R→S) P

5. B P – want S

6. A 2,5 →O

7. C 3,6 →O

8. R → S 4,7 →O

Chapter 1 - Conditionals

Dona Warren, Department of Philosophy, The University of Wisconsin – Stevens Point 61

9. S 1,8 →O

Valid

112

4. L → (P → Q), S→R, P, L, Q→R |- S

1. L → (P → Q) P

2. S→R P

4. P P

5. L P

6. Q → R P - want S

7. P → Q 1,5 →O

8. Q 4,7 →O

9. R 6,8 →O

10. S 2,9 FAC!

Invalid

113

5. Assess mentally: L→M, (L→M) → (A→B), B | A

L→M and (L→M)→(A→B) gives us A→B. But to go from A→B and B to A is to commit

the Fallacy of Assuming the Consequent.

Invalid.

114

6. Assess mentally: L→M, (L→M) → (A→B), A | B

L→M and (L→M)→(A→B) gives us A→B. And A→B and A gives us B.

Valid.

115

7. Write the symbolization and assess mentally: “If God exists then the world would be well-

designed. And the world is well designed. Therefore God exists.”

G→W, W |- G

From to go from G→W and W to G is to commit the Fallacy of Assuming the Consequent.

Invalid.

Written out, the “proof” is:

1. G→W P

2. W P – want G

Chapter 1 - Conditionals

Dona Warren, Department of Philosophy, The University of Wisconsin – Stevens Point 62

3. G 1, 2 FAC!

116

8. Write the symbolization and assess mentally: “If the world is well-designed then God exists.

And the world is well designed. Therefore God exists.”

W→G, W |- G

From W→G and W, we can conclude G.

Valid.

Written out, the proof is:

1. W→G P

2. W P – want G

3. G 1, 2 →O

117

9. Write the symbolization and assess mentally: “People have an innate sense of right and

wrong. People can have an innate sense of right and wrong only if God exists. And provided

that God exists, evil-doers will be punished. Therefore, evil-doers will be punished.”

I, I→G, G→P |- P

From I and I→G we get G. From G and G→P we get P.

Valid

Written out, the proof is:

1. I P

2. I→G P

3. G→P P – want P

4. G 1, 2 →O

5. P 3, 4 →O

118

10. Write the symbolization and assess mentally: “People have an innate sense of right and

wrong. People can have an innate sense of right and wrong if God exists. And provided that

God exists, evil-doers will be punished. Therefore, evil-doers will be punished.”

I, G→I, G→P |- P

To from I and G→I to G is to commit the Fallacy of Assuming the Consequent. From G and

G→P we can get P, but that doesn’t matter because we can’t validly get G in the first place.

Chapter 1 - Conditionals

Dona Warren, Department of Philosophy, The University of Wisconsin – Stevens Point 63

Invalid

Written out, the “proof” is:

1. I P

2. G→I P

3. G→P P – want P

4. G 1, 2 FAC!

5. P 3, 4 →O

119

11. “If lying can be morally permissible provided that it sometimes serves to promote the

greatest happiness for the greatest number of people then utilitarianism is right. In fact,

utilitarianism is right. And lying does sometimes serve to promote the greatest happiness for

the greatest number of people. Therefore, lying can be morally permissible.”

(G → P) → U, U, G |- P

1. (G → P) → U P

2. U P

3. G P – want P

4. G → P 1, 2 FAC!

5. P 3, 4 →O

Invalid

120

12. “If utilitarianism is right then lying can be morally permissible provided that it sometimes

serves to promote the greatest happiness for the greatest number of people. In fact,

utilitarianism is right. And lying does sometimes serve to promote the greatest happiness for

the greatest number of people. Therefore, lying can be morally permissible.”

U → (G → P), U, G |- P

1. U → (G → P) P

2. U P

3. G P – want P

4. G → P 1, 2 →O

5. P 3, 4 →O

Valid